Difference between revisions of "2011 AMC 12A Problems/Problem 8"

(Solution)
m
 
(23 intermediate revisions by 9 users not shown)
Line 9: Line 9:
 
\textbf{(E)}\ 43 </math>
 
\textbf{(E)}\ 43 </math>
  
== Solution ==
+
==Solution 1==
===Solution 1===
 
 
Let <math>A=x</math>. Then from <math>A+B+C=30</math>, we find that <math>B=25-x</math>. From <math>B+C+D=30</math>, we then get that <math>D=x</math>. Continuing this pattern, we find <math>E=25-x</math>, <math>F=5</math>, <math>G=x</math>, and finally <math>H=25-x</math>. So <math>A+H=x+25-x=25 \rightarrow \boxed{\textbf{C}}</math>
 
Let <math>A=x</math>. Then from <math>A+B+C=30</math>, we find that <math>B=25-x</math>. From <math>B+C+D=30</math>, we then get that <math>D=x</math>. Continuing this pattern, we find <math>E=25-x</math>, <math>F=5</math>, <math>G=x</math>, and finally <math>H=25-x</math>. So <math>A+H=x+25-x=25 \rightarrow \boxed{\textbf{C}}</math>
  
  
===Solution 2===
+
==Solution 2==
 
Given that the sum of 3 consecutive terms is 30, we have
 
Given that the sum of 3 consecutive terms is 30, we have
 
<math>(A+B+C)+(C+D+E)+(F+G+H)=90</math> and <math>(B+C+D)+(E+F+G)=60</math>
 
<math>(A+B+C)+(C+D+E)+(F+G+H)=90</math> and <math>(B+C+D)+(E+F+G)=60</math>
Line 23: Line 22:
  
  
===Solution 3 (the tedious one)===
+
==Solution 3 (the tedious one)==
From the given information, we can deduct the following equations:
+
From the given information, we can deduce the following equations:
  
 
<math>A+B=25, B+D=25, D+E=25, D+E+F=30, E+F+G
 
<math>A+B=25, B+D=25, D+E=25, D+E+F=30, E+F+G
 
=30</math>, and <math>F+G+H=30</math>.
 
=30</math>, and <math>F+G+H=30</math>.
  
 
+
We can then cleverly manipulate the equations above by adding and subtracting them to be left with the answer.
We can then add and subtract the equations above to be left with the answer.
 
  
 
<math>(A+B)-(B+D)=25-25 \implies (A-D)=0</math>
 
<math>(A+B)-(B+D)=25-25 \implies (A-D)=0</math>
  
<math>(A-D)+(D+E)=0+25 \implies (A+E)=25</math>
+
<math>(A-D)+(D+E)=0+25 \implies (A+E)=25</math>  
  
<math>(A+E)-(E+F+G)=25-30 \implies (A-F-G)=-5</math>
+
<math>(A+E)-(E+F+G)=25-30 \implies (A-F-G)=-5</math> (Notice how we don't use <math>D+E+F=30</math>)
  
 
<math>(A-F-G)+(F+G+H)=-5+30 \implies (A+H)=25</math>
 
<math>(A-F-G)+(F+G+H)=-5+30 \implies (A+H)=25</math>
  
Therefore, we have that <math>A+H=25 \rightarrow \boxed{\textbf{C}}</math>
+
Therefore, we have <math>A+H=25 \rightarrow \boxed{\textbf{C}}</math>
 +
 
 +
~JinhoK
 +
 
 +
== Solution 4 (the cheap one) ==
 +
 
 +
Since all of the answer choices are constants, it shouldn't matter what we pick <math>A</math> and <math>B</math> to be, so let <math>A = 20</math> and <math>B = 5</math>. Then <math>D = 30 - B -C = 20</math>, <math>E = 30 - D - C = 5</math>, <math>F = 30 - D - E =5</math>, and so on until we get <math>H = 5</math>. Thus <math>A + H = \boxed{\mathbf{(C)}25}</math>
 +
 
 +
== Solution 5 (assumption) ==
 +
 
 +
Assume the sequence is <cmath>15,10,5,15,10,5,15,10</cmath>.
 +
 
 +
Thus, <math>15+10=\boxed{25}</math> or option <math>\boxed{\mathbf{(C )}25}</math>
 +
 
 +
~SirAppel
 +
 
 +
== Solution 6 ==
 +
 
 +
Notice that the period of the sequence is <math>3</math> as given. (If this isn't clear we can show an example: <math>A+B+C=B+C=D</math> <math>\Leftrightarrow</math> <math>A=D</math>). Then <math>A=D</math> and <math>H=B</math>, so <math>A+H=D+B=D+B+C-C=30-5=\boxed{\textbf{(C)}\;25}</math>.
 +
 
 +
~eevee9406
 +
 
 +
==Note==
 +
Something useful to shorten a lot of the solutions above is to notice <cmath>5 + D + E = D + E + F</cmath> so F = 5
 +
 
 +
==Video Solution==
 +
 
 +
https://www.youtube.com/watch?v=6tlqpAcmbz4
 +
~Shreyas S
 +
 
 +
==Podcast Solution==
 +
 
 +
https://www.buzzsprout.com/56982/episodes/383730 (Episode starts with a solution to this question)
 +
—[[User:wescarroll|wescarroll]]
  
 
== See also ==
 
== See also ==
 
{{AMC12 box|year=2011|num-b=7|num-a=9|ab=A}}
 
{{AMC12 box|year=2011|num-b=7|num-a=9|ab=A}}
 +
{{AMC10 box|year=2011|num-b=16|num-a=18|ab=A}}
 
{{MAA Notice}}
 
{{MAA Notice}}

Latest revision as of 23:12, 8 April 2024

Problem

In the eight term sequence $A$, $B$, $C$, $D$, $E$, $F$, $G$, $H$, the value of $C$ is $5$ and the sum of any three consecutive terms is $30$. What is $A+H$?

$\textbf{(A)}\ 17 \qquad \textbf{(B)}\ 18 \qquad \textbf{(C)}\ 25 \qquad \textbf{(D)}\ 26 \qquad \textbf{(E)}\ 43$

Solution 1

Let $A=x$. Then from $A+B+C=30$, we find that $B=25-x$. From $B+C+D=30$, we then get that $D=x$. Continuing this pattern, we find $E=25-x$, $F=5$, $G=x$, and finally $H=25-x$. So $A+H=x+25-x=25 \rightarrow \boxed{\textbf{C}}$


Solution 2

Given that the sum of 3 consecutive terms is 30, we have $(A+B+C)+(C+D+E)+(F+G+H)=90$ and $(B+C+D)+(E+F+G)=60$

It follows that $A+B+C+D+E+F+G+H=85$ because $C=5$.

Subtracting, we have that $A+H=25\rightarrow \boxed{\textbf{C}}$.


Solution 3 (the tedious one)

From the given information, we can deduce the following equations:

$A+B=25, B+D=25, D+E=25, D+E+F=30, E+F+G =30$, and $F+G+H=30$.

We can then cleverly manipulate the equations above by adding and subtracting them to be left with the answer.

$(A+B)-(B+D)=25-25 \implies (A-D)=0$

$(A-D)+(D+E)=0+25 \implies (A+E)=25$

$(A+E)-(E+F+G)=25-30 \implies (A-F-G)=-5$ (Notice how we don't use $D+E+F=30$)

$(A-F-G)+(F+G+H)=-5+30 \implies (A+H)=25$

Therefore, we have $A+H=25 \rightarrow \boxed{\textbf{C}}$

~JinhoK

Solution 4 (the cheap one)

Since all of the answer choices are constants, it shouldn't matter what we pick $A$ and $B$ to be, so let $A = 20$ and $B = 5$. Then $D = 30 - B -C = 20$, $E = 30 - D - C = 5$, $F = 30 - D - E =5$, and so on until we get $H = 5$. Thus $A + H = \boxed{\mathbf{(C)}25}$

Solution 5 (assumption)

Assume the sequence is \[15,10,5,15,10,5,15,10\].

Thus, $15+10=\boxed{25}$ or option $\boxed{\mathbf{(C )}25}$

~SirAppel

Solution 6

Notice that the period of the sequence is $3$ as given. (If this isn't clear we can show an example: $A+B+C=B+C=D$ $\Leftrightarrow$ $A=D$). Then $A=D$ and $H=B$, so $A+H=D+B=D+B+C-C=30-5=\boxed{\textbf{(C)}\;25}$.

~eevee9406

Note

Something useful to shorten a lot of the solutions above is to notice \[5 + D + E = D + E + F\] so F = 5

Video Solution

https://www.youtube.com/watch?v=6tlqpAcmbz4 ~Shreyas S

Podcast Solution

https://www.buzzsprout.com/56982/episodes/383730 (Episode starts with a solution to this question) —wescarroll

See also

2011 AMC 12A (ProblemsAnswer KeyResources)
Preceded by
Problem 7
Followed by
Problem 9
1 2 3 4 5 6 7 8 9 10 11 12 13 14 15 16 17 18 19 20 21 22 23 24 25
All AMC 12 Problems and Solutions
2011 AMC 10A (ProblemsAnswer KeyResources)
Preceded by
Problem 16
Followed by
Problem 18
1 2 3 4 5 6 7 8 9 10 11 12 13 14 15 16 17 18 19 20 21 22 23 24 25
All AMC 10 Problems and Solutions

The problems on this page are copyrighted by the Mathematical Association of America's American Mathematics Competitions. AMC logo.png